Maximal ideal in ring of power series












3












$begingroup$


If $R$ is a commutative ring with identity we know that the maximal ideals of the ring of power series over $R$ have the form $M’=(M,x)$ where $M$ is a maximal ideal of $R$. Do you have a counterexample that shows that if $R$ doesn’t have an identity then the theorem doesn’t hold?
I really don’t have any idea where or how to start.
Reference: Burton’s “First course in ring and ideals” page 117 theorem 7-4










share|cite|improve this question









$endgroup$












  • $begingroup$
    You mean you want a ring $R$ without identity such that $R[[x]]$ has a maximal ideal that's not of that form? In rings without identity, authors usually require more of an ideal to consider it "maximal". For instance, they sometimes require it to be "modular." Is there any such requirement or is it just "a proper ideal maximal among proper ideals."
    $endgroup$
    – rschwieb
    Dec 18 '18 at 20:26












  • $begingroup$
    I am just now considering what $R[x]$ and $R[[x]]$ look like when $R^2={0}$... it seems... interesting, if you define it so that $xin R[x]$
    $endgroup$
    – rschwieb
    Dec 18 '18 at 20:30












  • $begingroup$
    Is this an actual question that was posed or is it something less formal? it's pretty uncommon for algebraists to be interested in such a situation.
    $endgroup$
    – rschwieb
    Dec 18 '18 at 20:37












  • $begingroup$
    I’m only searching a counterexample to this theorem. I thought that searching among ring without identity was easier than searching among noncommutative ring. However I don’t know what it means to be modular.
    $endgroup$
    – MathIsLove
    Dec 18 '18 at 20:42






  • 2




    $begingroup$
    Yes, for me a maximal ideal is just a maximal ideal among proper ideali. If R isn’t commutative, the maximal ideal is right. For the last question, I mean what you say.
    $endgroup$
    – MathIsLove
    Dec 18 '18 at 21:00
















3












$begingroup$


If $R$ is a commutative ring with identity we know that the maximal ideals of the ring of power series over $R$ have the form $M’=(M,x)$ where $M$ is a maximal ideal of $R$. Do you have a counterexample that shows that if $R$ doesn’t have an identity then the theorem doesn’t hold?
I really don’t have any idea where or how to start.
Reference: Burton’s “First course in ring and ideals” page 117 theorem 7-4










share|cite|improve this question









$endgroup$












  • $begingroup$
    You mean you want a ring $R$ without identity such that $R[[x]]$ has a maximal ideal that's not of that form? In rings without identity, authors usually require more of an ideal to consider it "maximal". For instance, they sometimes require it to be "modular." Is there any such requirement or is it just "a proper ideal maximal among proper ideals."
    $endgroup$
    – rschwieb
    Dec 18 '18 at 20:26












  • $begingroup$
    I am just now considering what $R[x]$ and $R[[x]]$ look like when $R^2={0}$... it seems... interesting, if you define it so that $xin R[x]$
    $endgroup$
    – rschwieb
    Dec 18 '18 at 20:30












  • $begingroup$
    Is this an actual question that was posed or is it something less formal? it's pretty uncommon for algebraists to be interested in such a situation.
    $endgroup$
    – rschwieb
    Dec 18 '18 at 20:37












  • $begingroup$
    I’m only searching a counterexample to this theorem. I thought that searching among ring without identity was easier than searching among noncommutative ring. However I don’t know what it means to be modular.
    $endgroup$
    – MathIsLove
    Dec 18 '18 at 20:42






  • 2




    $begingroup$
    Yes, for me a maximal ideal is just a maximal ideal among proper ideali. If R isn’t commutative, the maximal ideal is right. For the last question, I mean what you say.
    $endgroup$
    – MathIsLove
    Dec 18 '18 at 21:00














3












3








3


0



$begingroup$


If $R$ is a commutative ring with identity we know that the maximal ideals of the ring of power series over $R$ have the form $M’=(M,x)$ where $M$ is a maximal ideal of $R$. Do you have a counterexample that shows that if $R$ doesn’t have an identity then the theorem doesn’t hold?
I really don’t have any idea where or how to start.
Reference: Burton’s “First course in ring and ideals” page 117 theorem 7-4










share|cite|improve this question









$endgroup$




If $R$ is a commutative ring with identity we know that the maximal ideals of the ring of power series over $R$ have the form $M’=(M,x)$ where $M$ is a maximal ideal of $R$. Do you have a counterexample that shows that if $R$ doesn’t have an identity then the theorem doesn’t hold?
I really don’t have any idea where or how to start.
Reference: Burton’s “First course in ring and ideals” page 117 theorem 7-4







abstract-algebra maximal-and-prime-ideals






share|cite|improve this question













share|cite|improve this question











share|cite|improve this question




share|cite|improve this question










asked Dec 18 '18 at 19:54









MathIsLoveMathIsLove

214




214












  • $begingroup$
    You mean you want a ring $R$ without identity such that $R[[x]]$ has a maximal ideal that's not of that form? In rings without identity, authors usually require more of an ideal to consider it "maximal". For instance, they sometimes require it to be "modular." Is there any such requirement or is it just "a proper ideal maximal among proper ideals."
    $endgroup$
    – rschwieb
    Dec 18 '18 at 20:26












  • $begingroup$
    I am just now considering what $R[x]$ and $R[[x]]$ look like when $R^2={0}$... it seems... interesting, if you define it so that $xin R[x]$
    $endgroup$
    – rschwieb
    Dec 18 '18 at 20:30












  • $begingroup$
    Is this an actual question that was posed or is it something less formal? it's pretty uncommon for algebraists to be interested in such a situation.
    $endgroup$
    – rschwieb
    Dec 18 '18 at 20:37












  • $begingroup$
    I’m only searching a counterexample to this theorem. I thought that searching among ring without identity was easier than searching among noncommutative ring. However I don’t know what it means to be modular.
    $endgroup$
    – MathIsLove
    Dec 18 '18 at 20:42






  • 2




    $begingroup$
    Yes, for me a maximal ideal is just a maximal ideal among proper ideali. If R isn’t commutative, the maximal ideal is right. For the last question, I mean what you say.
    $endgroup$
    – MathIsLove
    Dec 18 '18 at 21:00


















  • $begingroup$
    You mean you want a ring $R$ without identity such that $R[[x]]$ has a maximal ideal that's not of that form? In rings without identity, authors usually require more of an ideal to consider it "maximal". For instance, they sometimes require it to be "modular." Is there any such requirement or is it just "a proper ideal maximal among proper ideals."
    $endgroup$
    – rschwieb
    Dec 18 '18 at 20:26












  • $begingroup$
    I am just now considering what $R[x]$ and $R[[x]]$ look like when $R^2={0}$... it seems... interesting, if you define it so that $xin R[x]$
    $endgroup$
    – rschwieb
    Dec 18 '18 at 20:30












  • $begingroup$
    Is this an actual question that was posed or is it something less formal? it's pretty uncommon for algebraists to be interested in such a situation.
    $endgroup$
    – rschwieb
    Dec 18 '18 at 20:37












  • $begingroup$
    I’m only searching a counterexample to this theorem. I thought that searching among ring without identity was easier than searching among noncommutative ring. However I don’t know what it means to be modular.
    $endgroup$
    – MathIsLove
    Dec 18 '18 at 20:42






  • 2




    $begingroup$
    Yes, for me a maximal ideal is just a maximal ideal among proper ideali. If R isn’t commutative, the maximal ideal is right. For the last question, I mean what you say.
    $endgroup$
    – MathIsLove
    Dec 18 '18 at 21:00
















$begingroup$
You mean you want a ring $R$ without identity such that $R[[x]]$ has a maximal ideal that's not of that form? In rings without identity, authors usually require more of an ideal to consider it "maximal". For instance, they sometimes require it to be "modular." Is there any such requirement or is it just "a proper ideal maximal among proper ideals."
$endgroup$
– rschwieb
Dec 18 '18 at 20:26






$begingroup$
You mean you want a ring $R$ without identity such that $R[[x]]$ has a maximal ideal that's not of that form? In rings without identity, authors usually require more of an ideal to consider it "maximal". For instance, they sometimes require it to be "modular." Is there any such requirement or is it just "a proper ideal maximal among proper ideals."
$endgroup$
– rschwieb
Dec 18 '18 at 20:26














$begingroup$
I am just now considering what $R[x]$ and $R[[x]]$ look like when $R^2={0}$... it seems... interesting, if you define it so that $xin R[x]$
$endgroup$
– rschwieb
Dec 18 '18 at 20:30






$begingroup$
I am just now considering what $R[x]$ and $R[[x]]$ look like when $R^2={0}$... it seems... interesting, if you define it so that $xin R[x]$
$endgroup$
– rschwieb
Dec 18 '18 at 20:30














$begingroup$
Is this an actual question that was posed or is it something less formal? it's pretty uncommon for algebraists to be interested in such a situation.
$endgroup$
– rschwieb
Dec 18 '18 at 20:37






$begingroup$
Is this an actual question that was posed or is it something less formal? it's pretty uncommon for algebraists to be interested in such a situation.
$endgroup$
– rschwieb
Dec 18 '18 at 20:37














$begingroup$
I’m only searching a counterexample to this theorem. I thought that searching among ring without identity was easier than searching among noncommutative ring. However I don’t know what it means to be modular.
$endgroup$
– MathIsLove
Dec 18 '18 at 20:42




$begingroup$
I’m only searching a counterexample to this theorem. I thought that searching among ring without identity was easier than searching among noncommutative ring. However I don’t know what it means to be modular.
$endgroup$
– MathIsLove
Dec 18 '18 at 20:42




2




2




$begingroup$
Yes, for me a maximal ideal is just a maximal ideal among proper ideali. If R isn’t commutative, the maximal ideal is right. For the last question, I mean what you say.
$endgroup$
– MathIsLove
Dec 18 '18 at 21:00




$begingroup$
Yes, for me a maximal ideal is just a maximal ideal among proper ideali. If R isn’t commutative, the maximal ideal is right. For the last question, I mean what you say.
$endgroup$
– MathIsLove
Dec 18 '18 at 21:00










1 Answer
1






active

oldest

votes


















1












$begingroup$

I don't know for sure if this suits your needs or not, but if $R=2mathbb Z/4mathbb Z$ and $M=(2x)lhd R[[x]]$, then $R[[x]]/Mcong R$ has two elements, so $M$ is maximal (in the sense you specified in the comments.)






share|cite|improve this answer









$endgroup$













  • $begingroup$
    I don’t think it is the couterexample I’m searching. Sorry. I think that I’m not able to explain what I want to say in English.
    $endgroup$
    – MathIsLove
    Dec 18 '18 at 21:21










  • $begingroup$
    @MathIsLove What condition did I not satisfy? (I don't mean it rhetorically, I'm asking seriously so I can check if I can improve it.)
    $endgroup$
    – rschwieb
    Dec 18 '18 at 21:22












  • $begingroup$
    I have to search a maximal ideal M’ in the ring of power series over R (R without identity) that doesn’t have the form M’=(M,x) where M is maximal in R. You didn’t showed that it can’t have that form but only that it is maximal.
    $endgroup$
    – MathIsLove
    Dec 18 '18 at 21:26










  • $begingroup$
    @MathIsLove It can't have that form because $xnotin R[[x]]$, in my case. And $(2x)cap R={0}$, so $M$ couldn't be anything but ${0}$.
    $endgroup$
    – rschwieb
    Dec 18 '18 at 21:28












  • $begingroup$
    Oh. It seems to work. Thanks!
    $endgroup$
    – MathIsLove
    Dec 18 '18 at 21:36












Your Answer





StackExchange.ifUsing("editor", function () {
return StackExchange.using("mathjaxEditing", function () {
StackExchange.MarkdownEditor.creationCallbacks.add(function (editor, postfix) {
StackExchange.mathjaxEditing.prepareWmdForMathJax(editor, postfix, [["$", "$"], ["\\(","\\)"]]);
});
});
}, "mathjax-editing");

StackExchange.ready(function() {
var channelOptions = {
tags: "".split(" "),
id: "69"
};
initTagRenderer("".split(" "), "".split(" "), channelOptions);

StackExchange.using("externalEditor", function() {
// Have to fire editor after snippets, if snippets enabled
if (StackExchange.settings.snippets.snippetsEnabled) {
StackExchange.using("snippets", function() {
createEditor();
});
}
else {
createEditor();
}
});

function createEditor() {
StackExchange.prepareEditor({
heartbeatType: 'answer',
autoActivateHeartbeat: false,
convertImagesToLinks: true,
noModals: true,
showLowRepImageUploadWarning: true,
reputationToPostImages: 10,
bindNavPrevention: true,
postfix: "",
imageUploader: {
brandingHtml: "Powered by u003ca class="icon-imgur-white" href="https://imgur.com/"u003eu003c/au003e",
contentPolicyHtml: "User contributions licensed under u003ca href="https://creativecommons.org/licenses/by-sa/3.0/"u003ecc by-sa 3.0 with attribution requiredu003c/au003e u003ca href="https://stackoverflow.com/legal/content-policy"u003e(content policy)u003c/au003e",
allowUrls: true
},
noCode: true, onDemand: true,
discardSelector: ".discard-answer"
,immediatelyShowMarkdownHelp:true
});


}
});














draft saved

draft discarded


















StackExchange.ready(
function () {
StackExchange.openid.initPostLogin('.new-post-login', 'https%3a%2f%2fmath.stackexchange.com%2fquestions%2f3045623%2fmaximal-ideal-in-ring-of-power-series%23new-answer', 'question_page');
}
);

Post as a guest















Required, but never shown

























1 Answer
1






active

oldest

votes








1 Answer
1






active

oldest

votes









active

oldest

votes






active

oldest

votes









1












$begingroup$

I don't know for sure if this suits your needs or not, but if $R=2mathbb Z/4mathbb Z$ and $M=(2x)lhd R[[x]]$, then $R[[x]]/Mcong R$ has two elements, so $M$ is maximal (in the sense you specified in the comments.)






share|cite|improve this answer









$endgroup$













  • $begingroup$
    I don’t think it is the couterexample I’m searching. Sorry. I think that I’m not able to explain what I want to say in English.
    $endgroup$
    – MathIsLove
    Dec 18 '18 at 21:21










  • $begingroup$
    @MathIsLove What condition did I not satisfy? (I don't mean it rhetorically, I'm asking seriously so I can check if I can improve it.)
    $endgroup$
    – rschwieb
    Dec 18 '18 at 21:22












  • $begingroup$
    I have to search a maximal ideal M’ in the ring of power series over R (R without identity) that doesn’t have the form M’=(M,x) where M is maximal in R. You didn’t showed that it can’t have that form but only that it is maximal.
    $endgroup$
    – MathIsLove
    Dec 18 '18 at 21:26










  • $begingroup$
    @MathIsLove It can't have that form because $xnotin R[[x]]$, in my case. And $(2x)cap R={0}$, so $M$ couldn't be anything but ${0}$.
    $endgroup$
    – rschwieb
    Dec 18 '18 at 21:28












  • $begingroup$
    Oh. It seems to work. Thanks!
    $endgroup$
    – MathIsLove
    Dec 18 '18 at 21:36
















1












$begingroup$

I don't know for sure if this suits your needs or not, but if $R=2mathbb Z/4mathbb Z$ and $M=(2x)lhd R[[x]]$, then $R[[x]]/Mcong R$ has two elements, so $M$ is maximal (in the sense you specified in the comments.)






share|cite|improve this answer









$endgroup$













  • $begingroup$
    I don’t think it is the couterexample I’m searching. Sorry. I think that I’m not able to explain what I want to say in English.
    $endgroup$
    – MathIsLove
    Dec 18 '18 at 21:21










  • $begingroup$
    @MathIsLove What condition did I not satisfy? (I don't mean it rhetorically, I'm asking seriously so I can check if I can improve it.)
    $endgroup$
    – rschwieb
    Dec 18 '18 at 21:22












  • $begingroup$
    I have to search a maximal ideal M’ in the ring of power series over R (R without identity) that doesn’t have the form M’=(M,x) where M is maximal in R. You didn’t showed that it can’t have that form but only that it is maximal.
    $endgroup$
    – MathIsLove
    Dec 18 '18 at 21:26










  • $begingroup$
    @MathIsLove It can't have that form because $xnotin R[[x]]$, in my case. And $(2x)cap R={0}$, so $M$ couldn't be anything but ${0}$.
    $endgroup$
    – rschwieb
    Dec 18 '18 at 21:28












  • $begingroup$
    Oh. It seems to work. Thanks!
    $endgroup$
    – MathIsLove
    Dec 18 '18 at 21:36














1












1








1





$begingroup$

I don't know for sure if this suits your needs or not, but if $R=2mathbb Z/4mathbb Z$ and $M=(2x)lhd R[[x]]$, then $R[[x]]/Mcong R$ has two elements, so $M$ is maximal (in the sense you specified in the comments.)






share|cite|improve this answer









$endgroup$



I don't know for sure if this suits your needs or not, but if $R=2mathbb Z/4mathbb Z$ and $M=(2x)lhd R[[x]]$, then $R[[x]]/Mcong R$ has two elements, so $M$ is maximal (in the sense you specified in the comments.)







share|cite|improve this answer












share|cite|improve this answer



share|cite|improve this answer










answered Dec 18 '18 at 21:14









rschwiebrschwieb

107k12103252




107k12103252












  • $begingroup$
    I don’t think it is the couterexample I’m searching. Sorry. I think that I’m not able to explain what I want to say in English.
    $endgroup$
    – MathIsLove
    Dec 18 '18 at 21:21










  • $begingroup$
    @MathIsLove What condition did I not satisfy? (I don't mean it rhetorically, I'm asking seriously so I can check if I can improve it.)
    $endgroup$
    – rschwieb
    Dec 18 '18 at 21:22












  • $begingroup$
    I have to search a maximal ideal M’ in the ring of power series over R (R without identity) that doesn’t have the form M’=(M,x) where M is maximal in R. You didn’t showed that it can’t have that form but only that it is maximal.
    $endgroup$
    – MathIsLove
    Dec 18 '18 at 21:26










  • $begingroup$
    @MathIsLove It can't have that form because $xnotin R[[x]]$, in my case. And $(2x)cap R={0}$, so $M$ couldn't be anything but ${0}$.
    $endgroup$
    – rschwieb
    Dec 18 '18 at 21:28












  • $begingroup$
    Oh. It seems to work. Thanks!
    $endgroup$
    – MathIsLove
    Dec 18 '18 at 21:36


















  • $begingroup$
    I don’t think it is the couterexample I’m searching. Sorry. I think that I’m not able to explain what I want to say in English.
    $endgroup$
    – MathIsLove
    Dec 18 '18 at 21:21










  • $begingroup$
    @MathIsLove What condition did I not satisfy? (I don't mean it rhetorically, I'm asking seriously so I can check if I can improve it.)
    $endgroup$
    – rschwieb
    Dec 18 '18 at 21:22












  • $begingroup$
    I have to search a maximal ideal M’ in the ring of power series over R (R without identity) that doesn’t have the form M’=(M,x) where M is maximal in R. You didn’t showed that it can’t have that form but only that it is maximal.
    $endgroup$
    – MathIsLove
    Dec 18 '18 at 21:26










  • $begingroup$
    @MathIsLove It can't have that form because $xnotin R[[x]]$, in my case. And $(2x)cap R={0}$, so $M$ couldn't be anything but ${0}$.
    $endgroup$
    – rschwieb
    Dec 18 '18 at 21:28












  • $begingroup$
    Oh. It seems to work. Thanks!
    $endgroup$
    – MathIsLove
    Dec 18 '18 at 21:36
















$begingroup$
I don’t think it is the couterexample I’m searching. Sorry. I think that I’m not able to explain what I want to say in English.
$endgroup$
– MathIsLove
Dec 18 '18 at 21:21




$begingroup$
I don’t think it is the couterexample I’m searching. Sorry. I think that I’m not able to explain what I want to say in English.
$endgroup$
– MathIsLove
Dec 18 '18 at 21:21












$begingroup$
@MathIsLove What condition did I not satisfy? (I don't mean it rhetorically, I'm asking seriously so I can check if I can improve it.)
$endgroup$
– rschwieb
Dec 18 '18 at 21:22






$begingroup$
@MathIsLove What condition did I not satisfy? (I don't mean it rhetorically, I'm asking seriously so I can check if I can improve it.)
$endgroup$
– rschwieb
Dec 18 '18 at 21:22














$begingroup$
I have to search a maximal ideal M’ in the ring of power series over R (R without identity) that doesn’t have the form M’=(M,x) where M is maximal in R. You didn’t showed that it can’t have that form but only that it is maximal.
$endgroup$
– MathIsLove
Dec 18 '18 at 21:26




$begingroup$
I have to search a maximal ideal M’ in the ring of power series over R (R without identity) that doesn’t have the form M’=(M,x) where M is maximal in R. You didn’t showed that it can’t have that form but only that it is maximal.
$endgroup$
– MathIsLove
Dec 18 '18 at 21:26












$begingroup$
@MathIsLove It can't have that form because $xnotin R[[x]]$, in my case. And $(2x)cap R={0}$, so $M$ couldn't be anything but ${0}$.
$endgroup$
– rschwieb
Dec 18 '18 at 21:28






$begingroup$
@MathIsLove It can't have that form because $xnotin R[[x]]$, in my case. And $(2x)cap R={0}$, so $M$ couldn't be anything but ${0}$.
$endgroup$
– rschwieb
Dec 18 '18 at 21:28














$begingroup$
Oh. It seems to work. Thanks!
$endgroup$
– MathIsLove
Dec 18 '18 at 21:36




$begingroup$
Oh. It seems to work. Thanks!
$endgroup$
– MathIsLove
Dec 18 '18 at 21:36


















draft saved

draft discarded




















































Thanks for contributing an answer to Mathematics Stack Exchange!


  • Please be sure to answer the question. Provide details and share your research!

But avoid



  • Asking for help, clarification, or responding to other answers.

  • Making statements based on opinion; back them up with references or personal experience.


Use MathJax to format equations. MathJax reference.


To learn more, see our tips on writing great answers.




draft saved


draft discarded














StackExchange.ready(
function () {
StackExchange.openid.initPostLogin('.new-post-login', 'https%3a%2f%2fmath.stackexchange.com%2fquestions%2f3045623%2fmaximal-ideal-in-ring-of-power-series%23new-answer', 'question_page');
}
);

Post as a guest















Required, but never shown





















































Required, but never shown














Required, but never shown












Required, but never shown







Required, but never shown

































Required, but never shown














Required, but never shown












Required, but never shown







Required, but never shown







Popular posts from this blog

Plaza Victoria

In PowerPoint, is there a keyboard shortcut for bulleted / numbered list?

How to put 3 figures in Latex with 2 figures side by side and 1 below these side by side images but in...